Unit step function from piecewise

Click For Summary
The discussion revolves around finding the Laplace transform of a piecewise function defined by specific intervals. The function is expressed using the unit step function, with adjustments made to ensure the correct form for transformation. The user initially struggled with the last inequality but resolved it by modifying the expression to fit the required format for the Laplace transform. The final Laplace transform is derived using the properties of the unit step function and the function's intervals. The main focus is on correctly applying the Laplace transform to the piecewise function using the unit step function.
cdotter
Messages
305
Reaction score
0

Homework Statement


<br /> f(x) = \begin{cases}<br /> 0, &amp; t &lt; \pi \\<br /> t - \pi , &amp; \pi \leq t &lt; 2 \pi \\<br /> 0, &amp; t \geq 2 \pi<br /> \end{cases}<br />

Homework Equations


Unit step function:
u_c(t) = \begin{cases}<br /> 0, &amp; t &lt; c \\<br /> 1 , &amp; t \geq c \\<br /> \end{cases}<br />

The Attempt at a Solution



u_{\pi}(t)(t-\pi) - u_{2 \pi}(t)(t-2 \pi) gives me:

<br /> f(x) = \begin{cases}<br /> 0, &amp; t &lt; \pi \\<br /> t - \pi , &amp; \pi \leq t &lt; 2 \pi \\<br /> \pi, &amp; t \geq 2 \pi<br /> \end{cases}<br />

I'm not sure what to do to get the last inequality correct?
 
Physics news on Phys.org
Is u_{\pi}(t)(t-\pi) - u_{2 \pi}(t)(t- \pi) allowed? That does work but the "c" doesn't match.
 
I figured it out. :smile: It is allowed, it just needs forcing.

u_{\pi}(t)(t-\pi) - u_{2 \pi}(t)(t- \pi) \rightarrow u_{\pi}(t)(t-\pi) - u_{2 \pi}(t)((t- \pi) - \pi)

Now that it's in the form u_c(t)(t-c), the Laplace transform can be taken:

<br /> \begin{aligned}<br /> F(s) &amp;= L\{u_{\pi}t(t- \pi)-u_{2 \pi}(t)((t- \pi)- \pi)\}\\<br /> F(s) &amp;= e^{- \pi s}L\{t\} - e^{-2 \pi s}L\{t- \pi\}\\<br /> F(s) &amp;= \frac{e^{- \pi s}}{s^2} - e^{- 2 \pi s} \cdot (\frac{1}{s^2} - \frac{\pi}{s})<br /> \end{aligned}<br />
 
What the exact wording of the question?
 
SammyS said:
What the exact wording of the question?

The question asked to take the Laplace transform of the piecewise function. I had to create a step function and use L\{u_c(t)f(t-c)\} = e^{-cs}L\{f(t)\}, which I figured out.
 
Question: A clock's minute hand has length 4 and its hour hand has length 3. What is the distance between the tips at the moment when it is increasing most rapidly?(Putnam Exam Question) Answer: Making assumption that both the hands moves at constant angular velocities, the answer is ## \sqrt{7} .## But don't you think this assumption is somewhat doubtful and wrong?

Similar threads

  • · Replies 11 ·
Replies
11
Views
3K
  • · Replies 1 ·
Replies
1
Views
1K
Replies
4
Views
2K
  • · Replies 3 ·
Replies
3
Views
2K
  • · Replies 1 ·
Replies
1
Views
1K
  • · Replies 1 ·
Replies
1
Views
2K
  • · Replies 3 ·
Replies
3
Views
2K
  • · Replies 14 ·
Replies
14
Views
2K
  • · Replies 12 ·
Replies
12
Views
2K
Replies
8
Views
2K